Answer:
12 + 4x
Step-by-step explanation:
You take all of the ones which you can group together and so, you can take the 5+7 and make it 12 and the 2x + 2x = 4x. I hope this helps!
Which graph is correct?
The graph of the inequality y ≥ (1/2)x - 1 and x - y > 1 is attached. Shannon's graph is correct.
What is an equation?An equation is an expression that shows the relationship between two or more numbers and variables. Equations can either be linear, quadratic, cubic and so on depending on the degree.
Inequalities are used for the non equal comparison of numbers and variables.
Given the inequalities:
y ≥ (1/2)x - 1 (1)
and
x - y > 1 (2)
The graph of the inequality is attached. Shannon's graph is correct.
Find out more on equation at: https://brainly.com/question/2972832
#SPJ1
At the beginning of the year, Gabriel had $25 in savings and saved an additional $14
each week thereafter. Adrian started the year with $80 and saved $9 every week. Let
G represent the amount of money Gabriel has saved t weeks after the beginning of
the year and let A represent the amount of money Adrian has saved t weeks after the
beginning of the year. Write an equation for each situation, in terms of t, and
determine the number of weeks after the beginning of the year until Gabriel and
Adrian have the same amount of money saved.
Answer: 11 weeks
Step-by-step explanation:
G(t) = 25 + 14t
A(t) = 80 + 9t
set G equal to A and find t
25 + 14t = 80 + 9t
5t = 55
t = 11
Find the horizontal and vertical asymptotes of the function y = f(x) and use limits to justify your answers f(x) = 6x+5/3x-2
The given function is: `y = f(x) = 6x + 5/3x - 2`, the horizontal and vertical asymptotes of the function y = f(x) are 2 & 2/3 respectively.
Horizontal Asymptotes:
To find horizontal asymptotes, let's consider `x` is going to infinity and negative infinity.
Limit of f(x) as `x` approaches `infinity`:
We have to divide the numerator and denominator by the highest degree term of the denominator.
Here the highest degree term of the denominator is `x` so, dividing by `x`, we get:`
y = f(x)
= 6x/x + 5/x ÷ 3x/x - 2/x
On simplifying the above expression we get,
y = f(x)
= 6 + 5/x ÷ 3 - 2/x
As x goes to infinity, 5/x and 2/x tend to zero.
So, we get:
y = f(x)
= 6/3
= 2
Therefore, the horizontal asymptote is y = 2.
Limit of f(x) as `x` approaches negative `infinity`:
We have to divide the numerator and denominator by the highest degree term of the denominator.
Here the highest degree term of the denominator is `x` so, dividing by `x`, we get:
y = f(x)
= 6x/x + 5/x ÷ 3x/x - 2/x
On simplifying the above expression we get,
y = f(x)
= 6 + 5/x ÷ 3 - 2/x
As x goes to negative infinity, 5/x and 2/x tend to zero.
So, we get:
y = f(x)
= 6/3
= 2
Therefore, the horizontal asymptote is y = 2.
Vertical Asymptotes:
Vertical asymptotes occur at the values of `x` that make the denominator zero.
So, 3x - 2 = 0
x = 2/3.
So, x = 2/3 is the vertical asymptote of the given function.
To know more about vertical asymptotes visit:
https://brainly.com/question/29277484
#SPJ11
the market price of an article is rs 6000 13 percentage vat is levied after allowing 15 percent discount an the article what will be the price of the article find it also find the amount of vat
Answer:
\( \boxed{ \bold{ \sf{Price \: of \: the \: article = \: Rs \: 6678.3}}}\)
\( \boxed{ \bold{ \sf{Vat \: amount = \: \: Rs \: 768.3}}}\)
Step-by-step explanation:
Given,
Marked Price ( MP ) = Rs 6000
Vat percent = 13 %
Discount percent = 15%
▪️Finding the discount amount
Discount amount = Discount % of MP
= 15 % of 6000
= Rs 900
▪️Finding the selling price
Selling price ( SP )= Marked price - Discount amount
= Rs 6000 - Rs 900
= Rs 5910
▪️Finding the vat amount
Vat amount = Vat % of SP
= 13 % of 5910
= Rs 768.3
▪️Finding SP with VAT
SP with vat = SP + vat amount
= Rs 5910 + Rs 768.3
= Rs 6678.3
Hope I helped!
Best regards!
Sheila walks 3 miles in 9/10 of an hour. How fast does she walk per hour?
Answer:
1 hours = 60 min
60 x 9/10 = 54 min
3 divided by 54 = 1/18
so she walks 1/18 miles per hour.
Im not sure though. It might be 3 divided by 9/10 = 3 1/3. I’m not that confident in my answer. But it is definitely one of them, either 1/18 or 3 1/3.
Mandy bought a desktop computer system to start her business from home for $4,995. It is expected to depreciate at a rate of 10% per year. How much will her home computer system be worth after 9 years? Round to the nearest hundredth
Mandy's home computer system is expected to be worth $1,576.11.
Mandy's home computer system is expected to depreciate at a rate of 10% per year. After 1 year, the value of the computer system will be 90% of its original value.
After 2 years, it will be worth 90% of that value, or 0.9 × 0.9 = 0.81 times the original value. Continuing in this way, we can write the value of the computer system after n years as \(0.9^n\) times its original value. Thus, after 9 years, the computer system will be worth \(0.9^n\) times its original value:
Value after 9 years = 4995 × \(0.9^n\)
Using a calculator, we find that the value is approximately $1,576.11 when rounded to the nearest hundredth. Therefore, after 9 years, Mandy's home computer system is expected to be worth $1,576.11.
To know more about Round value, refer here:
https://brainly.com/question/30234919#
#SPJ11
A $100 pair of headphones is marked down by 10% and you also have a 20% off coupon. What is the final price, before tax?
Answer:
$72
Step-by-step explanation:
100 - 10% = 90
90 - 20% = 72
30% of 6th grade student made honor roll. if there are 248 students in 6th grade, how mant students were on the honor roll
Answer:
74.4
Step-by-step explanation:
Daniel buys a swing set priced at $788. If the sales tax is 7%, how much tax will Daniel pay? Submit
Answer:
he will pay 55.16
There are 30 Major League Baseball teams in the National League. Five of these teams will make the playoffs at the end of the season. The number of unique groups of teams that can make the playoffs is
Answer:
5
Step-by-step explanation:
Which step should be performed first when simplifying 18−3+11×2+5?
-3+11
2+5
18-3
11x2
Answer:
11 x 2
Step-by-step explanation:
By BODMAS rule,
B - Brackets
O - Orders
D - Division
M - Multiplication
A - Addition
S - Substraction
Consider DMAS alone.
Here, there is no division.
So,
Multiplication ( x ) should be the first step.
Use isometric dot paper to sketch the prism.
triangular prism 4 units high, with two sides of the base that are 3 units long and 4 units long
By using isometric dot paper in the picture we can see sketch the triangular prism.
Given that,
A triangular prism with height 4 units, with two sides of the base that are 3 units long and 4 units long
We have to use isometric dot paper to sketch a triangular prism.
We know that,
Mark the corner of the solid.
Draw 4 units down, 4 units to the left, and 3 units to the right.
Then draw a triangle for the top of the solid.
Draw segments 4 units down from each vertex for the vertical edges.
Connect the appropriate vertices using a dashed line for the hidden edge.
Therefore, by using isometric dot paper in the picture we can see sketch the triangular prism.
To know more about prism visit:
https://brainly.com/question/33888656
#SPJ4
1+(1/x+2)-(2x/x*x-4)=0
The value of x in 1 + (1/x+2) - (2x/x*x-4) = 0 are x = -2 and x = 3
How to determine the value of x in the equation?From the question, we have the following parameters that can be used in our computation:
1+(1/x+2)-(2x/x*x-4)=0
Rewrite as
1 + (1/x+2) - (2x/x*x-4) = 0
Rewrite the equation properly
So, we have
1 + (1/x+2) - (2x/x²- 4) = 0
Factorize the expression
So, we have
1 + (1/x+2) - (2x/(x- 2)(x + 2)) = 0
Multiply through the equation by x + 2
So, we have the following representation
x + 2 + 1 - 2x/(x- 2) = 0
Evaluate the like terms
So, we have the following representation
x + 3 - 2x/(x- 2) = 0
Multiply through the equation by x - 2
So, we have the following representation
x² - 2x + 3x - 6 - 2x = 0
So, we have
x² - x - 6 = 0
Factorize the equation
So, we have
x² + 2x - 3x - 6 = 0
This gives
x(x + 2) - 3(x + 2) = 0
So, we have
(x + 2)(x - 3)= 0
Evaluate
x = -2 and x = 3
Hence, the solution for x is x = -2 and x = 3
Read more about equations at
https://brainly.com/question/27887972
#SPJ1
Find an orthogonal basis of the subspace \( W \) of \( \mathbb{R}^{5} \) spanned by \[ \left\{\vec{u}_{1}=(1,1,1,0,1), \vec{u}_{2}=(1,0,0,-1,1), \vec{u}_{3}=(3,1,1,-2,3), \vec{u}_{4}=(0,2,1,1,-1)\righ
The resulting set of orthogonal unit vectors \(\(\{\vec{v}_{1}, \vec{v}_{2}, \vec{v}_{3}, \vec{v}_{4}\}\)\) forms an orthogonal basis for the subspace\(\(W\) of \(\mathbb{R}^{5}\).\)
To find an orthogonal basis of \(W\), we will use the Gram-Schmidt process to orthogonalize the given vectors \(\(\vec{u}_{1}\), \(\vec{u}_{2}\), \(\vec{u}_{3}\), and \(\vec{u}_{4}\).\)
1. Start by setting the first vector\(\(\vec{v}_{1} = \vec{u}_{1}\).\)
2. Orthogonalize the second vector \(\(\vec{u}_{2}\)\) with respect to \(\(\vec{v}_{1}\).\)Subtract the projection of \(\(\vec{u}_{2}\)\) onto \(\(\vec{v}_{1}\)\) from \\((\vec{u}_{2}\)\) to obtain an orthogonal vector \(\(\vec{v}_{2}\)\). Normalize \(\(\vec{v}_{2}\)\) to obtain an orthogonal unit vector.
3. Orthogonalize the third vector \(\(\vec{u}_{3}\)\) with respect to \(\(\vec{v}_{1}\) and \(\vec{v}_{2}\).\) Subtract the projections of\(\(\vec{u}_{3}\)\)onto \(\(\vec{v}_{1}\)\)and \(\(\vec{v}_{2}\)\) from \((\vec{u}_{3}\)\)to obtain an orthogonal vector \(\(\vec{v}_{3}\)\). Normalize \(\(\vec{v}_{3}\)\) to obtain an orthogonal unit vector.
4. Orthogonalize the fourth vector \(\(\vec{u}_{4}\)\) with respect to \(\(\vec{v}_{1}\), \\)\((\vec{v}_{2}\), and \(\vec{v}_{3}\).\) Subtract the projections of\(\(\vec{u}_{4}\) onto \(\vec{v}_{1}\), \\) \((\vec{v}_{2}\), and \(\vec{v}_{3}\) from \(\vec{u}_{4}\)\) to obtain an orthogonal vector \(\(\vec{v}_{4}\).\) Normalize \(\(\vec{v}_{4}\)\) to obtain an orthogonal unit vector.
Learn more about subspace here:
https://brainly.com/question/26727539
#SPJ11
Find an orthogonal basis of the subspace \(\( W \) of \( \mathbb{R}^{5} \)\)spanned by \(\[ \left\\)\({\vec{u}_{1}=(1,1,1,0,1), \vec{u}_{2}=(1,0,0,-1,1), \vec{u}_{3}=(3,1,1,-2,3), \vec{u}_{4}=(0,2,1,1,-1)\righ\)
help please middle school question will give brainlist
Answer:
-11, -6, 0, 3, 9, 10
Step-by-step explanation:
I don't really know how to explain but that is correct
Answer: -11,| -6 |, 0, | 3 |, 9, 10
Step-by-step explanation:
Determine if the following pair of statements are equivalent. (It is helpful to write the statements in symbolic form.) If you activate your cell phone before October 9, then you receive 100 free minutes. If you do not receive 100 free minutes, then you do not activate your cell phone before October 9.
The pair of statements can be written in symbolic form as follows:
P: You activate your cell phone before October 9.
Q: You receive 100 free minutes.
Statement 1: If P, then Q.
Statement 2: If not Q, then not P.
To determine if the statements are equivalent, we need to check if Statement 1 implies Statement 2 and if Statement 2 implies Statement 1.
If P, then Q: This means that if you activate your cell phone before October 9 (P), then you receive 100 free minutes (Q).
If not Q, then not P: This means that if you do not receive 100 free minutes (not Q), then you do not activate your cell phone before October 9 (not P).
The statements are indeed equivalent because they express the same logical relationship. If you activate your cell phone before October 9, you will receive 100 free minutes. Conversely, if you do not receive 100 free minutes, it means that you did not activate your cell phone before October 9.
Learn more about symbolic form here:
https://brainly.com/question/29296289
#SPJ11
Describe the horizontal cross section of the triangular pyramid.
A.triangle
B.rectangle
C.trapezoid
D.square
Why was algebre made?
Answer: to solve a problem and make a solution easier to find.
Algebra was made to solve mathematical problems that involve unknown quantities.
What is Equation?Two or more expressions with an Equal sign is called as Equation.
Algebra was developed to solve mathematical problems that involve unknown quantities.
It provides a way to represent and manipulate mathematical equations using symbols and rules, making it easier to solve complex problems and answer questions about relationships between variables.
The earliest known algebraic text is the "Al-jabr wa'l muqabalah" by the Persian mathematician Al-Khwarizmi, which was written in the 9th century.
Hence, Algebra was made to solve mathematical problems that involve unknown quantities.
To learn more on Equation:
https://brainly.com/question/10413253
#SPJ9
Does anybody know this? I’m kinda bad with graphing and I could really use an explanation on how to do this.
The equation for slope-intercept form is y=mx+b, where m is the slope and b is the y-intercept.
You can eliminate answer choices B, C, and D, because the y-intercept doesn't fit. The y-intercept is where the line crosses the y-axis. There's no way it could be 6 or -1/4.
So now it's between A and D. In the image, the line is going down so we know the slope is negative, and slope is rise/run. It goes down about 1 and to the right about 4.
Meaning A is the only option that makes sense.
Solve the system by substitution.
— 6x — 5y = 11
x = у
Answer:
(x, y) =(-1, - 1)
Step-by-step explanation:
-Substitute the value of x
-solve the equation
-Substitute the value of y
-check the possible solution
-check solution
-simplify
CAN SOMEONE PLEASE HELP ME WITH THIS STATISTICS PROBLEM I DONT UNDERSTAND IT AT ALLL!!!! The mean of a normal probability distribution is 390; the standard deviation is 95. a. Μ ± 1σ of the observations lie between what two values? Lower Value Upper Value b. Μ ± 2σ of the observations lie between what two values? Lower Value Upper Value c. Μ ± 3σ of the observations lie between what two values? Lower Value Upper Value
Answer:
A.)
Lower Value = 295
Upper Value = 485
B.)
Lower Value = 200
Upper Value = 580
C.)
Lower Value = 105
Upper Value = 675
Step-by-step explanation:
Given that mean (M) = 390
Standard deviation (σ) = 95
Lower Value = mean value - number of standard deviations specified
Upper Value = mean value + number of standard deviations specified
a. Μ ± 1σ of the observations lie between what two values?
Lower Value = 390 - 1(95) = 295
Upper Value = 390 + 1(95) = 485
b. Μ ± 2σ of the observations lie between what two values?
Lower Value = 390 - 2(95) = 200
Upper Value = 390 + 2(95) = 580
c. Μ ± 3σ of the observations lie between what two values?
Lower Value = 390 - 3(95) = 105
Upper Value = 390 + 3(95) = 675
Just need help with this one
Answer:
perimeter is 20 cm
Three component work in series. the component fail with probabilities p1=0.09, p2=0.11, and p3=0.28. what is the probability that the system will fail?
the probability that the system will fail is approximately 0.421096 or 42.11%.
To find the probability that the system will fail, we need to consider the components working in series. In this case, for the system to fail, at least one of the components must fail.
The probability of the system failing is equal to 1 minus the probability of all three components working together. Let's calculate it step by step:
1. Find the probability of all three components working together:
P(all components working) = (1 - p1) * (1 - p2) * (1 - p3)
= (1 - 0.09) * (1 - 0.11) * (1 - 0.28)
= 0.91 * 0.89 * 0.72
≈ 0.578904
2. Calculate the probability of the system failing:
P(system failing) = 1 - P(all components working)
= 1 - 0.578904
≈ 0.421096
Therefore, the probability that the system will fail is approximately 0.421096 or 42.11%.
Learn more about probability here
https://brainly.com/question/32117953
#SPJ4
Is the following figure a parallelogram? Why?
P
O No, opposite angles are congruent.
O Yes, opposite sides are congruent.
O No, opposite sides are congruent.
O Yes, opposite angles are congruent.
Sofia bought a package of 20 bottles of water. Each bottle contains w ounces. Write an
expression that shows the total number of ounces.
Answer:
20w
Step-by-step explanation:
If you have 20 bottles, and each contains w ounces, we can use the following example to solve.
If there were 4oz per bottle,
20 * 4 = 80
80oz
We have w oz per bottle,
20 * w = ?
20w
Let X0,X1,X2, . . . be independent identically distributed nonnegative random variables having a continuous distribution. Let N be the first index k for which Xk > X0. That is, N = 1 if X1 > X0,N = 2 if X1 ≤ X0 and X2 > X0, etc. Determine the probability mass function for N and the mean E[N]. (Interpretation: X0,X1, . . . are successive offers or bids on a car that you are trying to sell. Then, N is the index of the first bid that is better than the initial bid.)
The probability mass function for N is \(P(N = n) = (\frac{1}{2})^n\), and the mean E[N], is 0. This means that the expected value for the index of the first bid better than the initial bid, in this scenario, is 0.
What is the probability mass function?
The probability mass function (PMF) is a function that describes the probability distribution of a discrete random variable. In the case of N, the index of the first bid better than the initial bid, the PMF can be derived as follows:
\(P(N = n) = (\frac{1}{2})^n\).
To determine the probability mass function (PMF) for N and the mean E[N], let's analyze the problem step by step.
Given:
\(X_{0} ,X_{1}, X_{2} ,X_{3},...\) be independent identically distributed (\(\geq 0)\) random variables having a continuous distribution.N is the first index k for which \(X_{k} > X_{0}\).To find the PMF of N, we need to calculate the probability that N takes on a specific value n, where n is a positive integer.
Let's consider the event that N = n. This event occurs if\(X_{1} \leq X_{0}, X_{2} \leq X_{0},...,X_{(n-1)} \leq X_{0},X_{n} \leq X_{0}.\)
Since \(X_{0} ,X_{1}, X_{2} ,X_{3},...\)are identically distributed random variables, we can calculate the probability of each individual event using the properties of the continuous distribution. The probability that\(X_{k} > X_{0}\) for any specific k is given by:
\(P(X_{k} > X_{0})=\frac{1}{2}\) (assuming a symmetric continuous distribution)
Now, let's consider the event that \(X_{1} \leq X_{0}, X_{2} \leq X_{0},...,X_{(n-1)} \leq X_{0}.\)Since these events are independent, their probabilities:
\(P(X_{1} \leq X_{0}, X_{2} \leq X_{0},...,X_{(n-1)} \leq X_{0},X_{n} \leq X_{0})=[P(X_{1} \leq X_{0}]^{n-1}\)
Finally, the PMF of N is given by:
P(N = n) =\(P(X_{1} \leq X_{0}, X_{2} \leq X_{0},...,X_{(n-1)} \leq X_{0},X_{n} \leq X_{0})*P(X_{n} > X_{0})\\\\=[P(X_{1} \leq X_{0})]^{n-1}*P(X_{n} > X_{0})\\\\=(\frac{1}{2})^{n-1}*\frac{1}{2}\\\\=(\frac{1}{2})^n\)
So, the probability mass function (PMF) for N is\(P(N = n) = (\frac{1}{2})^n.\)
To calculate the mean E[N], we can use the formula for the expected value of a geometric distribution:
E[N] = ∑(n * P(N = n))
Since\(P(N = n) = (\frac{1}{2})^n.\), we have:
E[N] = ∑(\(n * (\frac{1}{2})^n\))
To calculate the sum, we can use the formula for the sum of an infinite geometric series:
E[N] = ∑(\(n * (\frac{1}{2})^n\))
= ∑(\(n * {x}^n\)) (where x = 1/2)
\(\frac{d}{dx}\sum(x^n) = \sum(n * x^{n-1})\)
Now, multiply both sides by x:
\(x\frac{d}{dx}\sum{x}^n = \sum(n * {x}^{n})\)
Substituting x = \(\frac{1}{2}\):
\(\frac{1}{2}*\frac{d}{dx}\sum(\frac{1}{2})^n = \sum(n * (\frac{1}{2})^{n})\)
The sum on the left side is a geometric series that converges to \(\frac{1}{1-x}\). So, we have:
\(\frac{1}{2}*\frac{d}{dx}(\frac{1}{1-\frac{1}{2}})=E[N]\\\)
Simplifying:
\(\frac{1}{2}*\frac{d}{dx}(\frac{1}{\frac{1}{2}})=E[N]\\\\\frac{1}{2}*\frac{d}{dx}(2)=E[N]\\\\\frac{1}{2}*0=E[N]\\\)
E[N] = 0
Therefore, the mean of N, E[N], is equal to 0.
To learn more about the probability mass function from the given link
brainly.com/question/30765833
#SPJ4
x/10 - 4 = 5
find X
plsplsplsplsplsplsplspls
Answer:
90
Step-by-step explanation:
X/10-4=5
add 4 to both sides
X/10=9
mulitiple 10 to both sides
X=90
check
(90)/10-4=5
9-4=5
Answer:
90
Step-by-step explanation:
complete the square to rewrite the following equation in standard form
By completing the square, the equation in standard form is (x - 2)² + (y + 4)² = 4².
What is the equation of a circle?In Mathematics and Geometry, the standard form of the equation of a circle is modeled by this mathematical equation;
(x - h)² + (y - k)² = r²
Where:
h and k represent the coordinates at the center of a circle.r represent the radius of a circle.From the information provided below, we have the following equation of a circle:
x² - 4x + y² + 8y = -4
x² - 4x + (-4/2)² + y² + 8y + (8/2)² = -4 + (-4/2)² + (8/2)²
x² - 4x + 4 + y² + 8y + 16 = -4 + 4 + 16
(x - 2)² + (y + 4)² = 16
(x - 2)² + (y + 4)² = 4²
Therefore, the center (h, k) is (2, -4) and the radius is equal to 4 units.
Read more on equation of a circle here: brainly.com/question/15626679
#SPJ4
Complete Question:
Complete the square to rewrite the following equation in standard form. x² - 4x + y² + 8y = -4.
a simple random sample of 64 concert tickets were drawn from a normal population. the mean and standard deviation of the sample were $120 and $25 respectivlly. estimate the population mean
95% confidence that the population means lies between $110.25 ($120 - $9.75) and $129.75 ($120 + $9.75).
To estimate the population mean, we can use the sample mean as an unbiased estimate. Therefore, the estimated population mean is also $120. However, to calculate the margin of error for this estimate, we can use the formula:
The margin of error = (z-score) x (standard deviation / square root of sample size)
Since the population is assumed to be normal, we can use the z-distribution. For a 95% confidence level, the z-score is 1.96. Substituting the given values, we get:
Margin of error = 1.96 x (25 / √64) = 9.75
Therefore, we can say with 95% confidence that the population mean lies between $110.25 ($120 - $9.75) and $129.75 ($120 + $9.75).
It's important to note that this is only an estimate and the true population mean could still be outside of this range. However, with a 95% confidence level, we can be fairly certain that our estimate is accurate within this range.
Learn more about margin of error here:
https://brainly.com/question/11832344
#SPJ11
let g be the function defined by g(x)=∫x−1(−12 cos(t3 2t))ⅆt for 0
The integral of g(x) is g(x) = -(1/2)sin(x³/2) + (1/2).
To find g(x), we use the following steps:
Evaluate the integral by applying the Fundamental Theorem of Calculus:Therefore, the integral of g(x) is g(x) = -(1/2)sin(x³/2) + (1/2).
Learn more about Integral here: brainly.com/question/18125359
#SPJ11